Đến nội dung

Hình ảnh

Topic bất đẳng thức THCS (2)


  • Chủ đề bị khóa Chủ đề bị khóa
Chủ đề này có 1115 trả lời

#201
alex_hoang

alex_hoang

    Thượng úy

  • Hiệp sỹ
  • 1152 Bài viết

Một bài thi học sinh giỏi
103. Cho a, b, c không âm thoả mãn : $ab + bc + ca = 1$. Chứng minh rằng $$\dfrac{1}{a + b} + \dfrac{1}{b + c} + \dfrac{1}{c + a} \ge \dfrac{5}{2}$$

Bài này cho ở THCS thì hơi ác,nó là một bài quen thộc nhưng đẹp mắt và hay .Nó cũng là một hệ quả của IRAN TST 1996.Xuất hiện trong nhiều tài liệu về BĐT bạn có thể xem trong cuốn STBĐT ở đây
http://diendantoanho...showtopic=61934
alex_hoang


HẸN NGÀY TRỞ LẠI VMF THÂN MẾN

http://www.scribd.co...oi-Ban-Cung-The

#202
Tham Lang

Tham Lang

    Thượng úy

  • Thành viên
  • 1149 Bài viết

Bài này cho ở THCS thì hơi ác,nó là một bài quen thộc nhưng đẹp mắt và hay .Nó cũng là một hệ quả của IRAN TST 1996.Xuất hiện trong nhiều tài liệu về BĐT bạn có thể xem trong cuốn STBĐT ở đây
http://diendantoanho...showtopic=61934

Giờ mình mới biết, STBDT thì chưa đọc nhiều nên mình không thấy :lol: (lười :)) Mình biết bài này vì đây là đề thi học sinh giỏi trường mình mà .

Off vĩnh viễn ! Không ngày trở lại.......


#203
Cao Xuân Huy

Cao Xuân Huy

    Thiếu úy

  • Hiệp sỹ
  • 592 Bài viết

Bài 97: Cho 3 số thực a,b,c. Chứng minh
$a^2+b^2+c^2\geq ab+bc+ac+\frac{(a-b)^2}{26}+\frac{(b-c)^2}{6}+\frac{(c-a)^2}{209}$

Biến đổi tương đương đúng là cả một nghệ thuật.

Ta có bất đẳng thức tương đương:
\[2{a^2} + 2{b^2} + 2{c^2} \ge 2ab + 2bc + 2ca + \frac{{{{(a - b)}^2}}}{{13}} + \frac{{{{(b - c)}^2}}}{3} + \frac{{2{{(c - a)}^2}}}{{209}}\]
\[ \Leftrightarrow {(a - b)^2} + {(b - c)^2} + {(c - a)^2} \ge \frac{{{{(a - b)}^2}}}{{13}} + \frac{{{{(b - c)}^2}}}{3} + \frac{{2{{(c - a)}^2}}}{{209}}\]
\[ \Leftrightarrow \frac{{12}}{{13}}{(a - b)^2} + \frac{2}{3}{(b - c)^2} + \frac{{207}}{{209}}{(a - b)^2} \ge 0\]

Bđt cuối cùng luôn đúng nên ta có ĐPCM.

Dấu "=" xảy ra khi $a=b=c$
________________________________________________________________________________

Tiếp tục với mấy bài này nhé

Bài 104: Cho các số thực $a,b,c$ sao cho $a \ge b \ge c >0$. Chứng minh:
\[\frac{1}{{{{(a + b)}^2}}} + \frac{1}{{{{(b + c)}^2}}} + \frac{1}{{{{(c + a)}^2}}} \ge \frac{2}{{(a + c)(b + c)}} + \frac{1}{{4ab}}\]

Bài 105: Cho các số thực dương $a,b,c$. Chứng minh:
\[\frac{1}{{a + 3b}} + \frac{1}{{b + 3c}} + \frac{1}{{c + 3a}} \ge \frac{1}{{a + 2b + c}} + \frac{1}{{b + 2c + a}} + \frac{1}{{c + 2a + b}}\]

Bài 106: Cho các số dương $a,b,c$ thỏa mãn $a+b+c=1$. Chứng minh:
\[\frac{1}{{4{a^2} + {b^2} + {c^2}}} + \frac{1}{{4{b^2} + {c^2} + {a^2}}} + \frac{1}{{4{c^2} + {a^2} + {b^2}}} \le \frac{9}{2}\]

Bài viết đã được chỉnh sửa nội dung bởi Cao Xuân Huy: 18-01-2012 - 08:34

Cao Xuân Huy tự hào là thành viên VMF

Hình đã gửi


#204
wallunint

wallunint

    Thượng sĩ

  • Thành viên
  • 273 Bài viết

Biến đổi tương đương đúng là cả một nghệ thuật.

Ta có bất đẳng thức tương đương:
$ 2{a^2} + 2{b^2} + 2{c^2} \ge 2ab + 2bc + 2ca + \frac{{{{(a - b)}^2}}}{{13}} + \frac{{{{(b - c)}^2}}}{3} + \frac{{2{{(c - a)}^2}}}{{209}} $
$ \Leftrightarrow {(a - b)^2} + {(b - c)^2} + {(c - a)^2} \ge \frac{{{{(a - b)}^2}}}{{13}} + \frac{{{{(b - c)}^2}}}{3} + \frac{{2{{(c - a)}^2}}}{{209}} $
$ \Leftrightarrow \frac{{12}}{{13}}{(a - b)^2} + \frac{2}{3}{(b - c)^2} + \frac{{207}}{{209}}{(a - b)^2} \ge 0 $

Bđt cuối cùng luôn đúng nên ta có ĐPCM.

Dấu "=" xảy ra khi $a=b=c$
________________________________________________________________________________

Tiếp tục với mấy bài này nhé

Bài 104: Cho các số thực $a,b,c$ sao cho $a \ge b \ge c >0$. Chứng minh:
$ \frac{1}{{{{(a + b)}^2}}} + \frac{1}{{{{(b + c)}^2}}} + \frac{1}{{{{(c + a)}^2}}} \ge \frac{2}{{(a + c)(b + c)}} + \frac{1}{{4ab}} $

Bài 105: Cho các số thực dương $a,b,c$. Chứng minh:
$ \frac{1}{{a + 3b}} + \frac{1}{{b + 3c}} + \frac{1}{{c + 3a}} \ge \frac{1}{{a + 2b + c}} + \frac{1}{{b + 2c + a}} + \frac{1}{{c + 2a + b}} $

Bài 106: Cho các số dương $a,b,c$ thỏa mãn $a+b+c=1$. Chứng minh:
$ \frac{1}{{4{a^2} + {b^2} + {c^2}}} + \frac{1}{{4{b^2} + {c^2} + {a^2}}} + \frac{1}{{4{c^2} + {a^2} + {b^2}}} \le \frac{9}{2} $


Bài 105: Áp dụng bđt AM-GM, ta có:

$ \frac{1}{{a + 3b}} + \frac{1}{{a + 2c + b}} \ge \frac{4}{{2\left( {a + 2b + c} \right)}} $
Chứng minh tương tự, ta có đpcm
Bài 104: Biến đổi tương đương và áp dụng bđt Am-GM, ta có:

$ \frac{1}{{\left( {a + b} \right)^2 }} + \frac{1}{{\left( {b + c} \right)^2 }} + \frac{1}{{\left( {c + a} \right)^2 }} \ge \frac{1}{{4ab}} + \frac{2}{{\left( {c + a} \right)\left( {b + c} \right)}} $

$ \Leftrightarrow \left( {\frac{1}{{\left( {c + a} \right)}} - \frac{1}{{\left( {b + c} \right)}}} \right)^2 \ge \frac{{\left( {a - b} \right)^2 }}{{4ab\left( {a + b} \right)^2 }} $
$ \Leftrightarrow 4ab\left( {a + b} \right)^2 \ge \left( {c + a} \right)^2 \left( {b + c} \right)^2 $
do đk bài toán $ \Rightarrow 4ab \ge 4b^2 \ge \left( {b + c} \right)^2 $ và $ \left( {a + b} \right)^2 \ge \left( {c + a} \right)^2 $
Đến đây, ta có bđt cần cm.

ZZ

Bài viết đã được chỉnh sửa nội dung bởi wallunint: 18-01-2012 - 09:16

Vì cuộc sống luôn thay màu .... !!!


#205
Ispectorgadget

Ispectorgadget

    Nothing

  • Quản lý Toán Phổ thông
  • 2946 Bài viết
Bài 107: Cho các số thực x,y,a $\geq -1$ thỏa mãn $\sqrt{1+x}+\sqrt{1+y}=2\sqrt{1+a}$. CMR:
$x+y\geq 2a$
ĐTTS lớp 10 chuyên Trần Đại Nghĩa - TPHCM 2003-2004

P/S: Walluint cho mình hỏi chữ ZZ là sao nhỉ :P

►|| The aim of life is self-development. To realize one's nature perfectly - that is what each of us is here for. ™ ♫


#206
nguyenta98

nguyenta98

    Thượng úy

  • Hiệp sỹ
  • 1259 Bài viết

Bài 107: Cho các số thực x,y,a $\geq -1$ thỏa mãn $\sqrt{1+x}+\sqrt{1+y}=2\sqrt{1+a}$. CMR:
$x+y\geq 2a$
ĐTTS lớp 10 chuyên Trần Đại Nghĩa - TPHCM 2003-2004

Bài 107:
Do $x,y,a\geq -1 \rightarrow 1+x,1+y,1+a\geq 0$
Từ phương trình ta có:
$\sqrt{(1+x)(1+a}+\sqrt{(1+y)(1+a)}=2(1+a)$
Lại có $\dfrac{1+x+1+a}{2}+\dfrac{1+y+1+a}{2}\geq \sqrt{(1+x)(1+a}+\sqrt{(1+y)(1+a)} \rightarrow \dfrac{1+x+1+a}{2}+\dfrac{1+y+1+a}{2}\geq 2(a+1) \leftrightarrow x+y\geq 2a \rightarrow Q.E.D$
Dấu $"=" \leftrightarrow x=y=a$

Bài viết đã được chỉnh sửa nội dung bởi nguyenta98: 18-01-2012 - 11:10


#207
Mai Duc Khai

Mai Duc Khai

    Thiếu úy

  • Thành viên
  • 617 Bài viết
Bài 108:
CMR: \[\left( {1 + {2^2}} \right)\left( {1 + {2^{{2^2}}}} \right)\left( {1 + {2^{{2^3}}}} \right).....\left( {1 + {2^{{2^n}}}} \right) < \frac{1}{3}{.2^{{2^{n + 1}}}}\]
Bài 109 :
Cho $a,b,c>0$: CMR
\[{\left( {a + b - c} \right)^n} + {\left( {b + c - a} \right)^n} + {\left( {c + a - b} \right)^n} \ge {a^n} + {b^n} + {c^n}\]

Tra cứu công thức toán trên diễn đàn


Học gõ Latex $\to$ Cách vẽ hình trên VMF


Điều mà mọi thành viên VMF cần phải biết và tuân thủ

______________________________________________________________________________________________

‎- Luật đời dạy em cách Giả Tạo
- Đời xô ... Em ngã
- Đời nham ... Em hiểm

- Đời chuyển ... Em xoay

Đời cay ... Em đắng


#208
HÀ QUỐC ĐẠT

HÀ QUỐC ĐẠT

    Thượng sĩ

  • Thành viên
  • 295 Bài viết
Bài 103:
BĐT$\Leftrightarrow 2(a+b+c)^{2}-5(a+b+c)+2+5abc\geq 0$(1)
Đặt p=a+b+c;q=ab+bc+ca,r=abc nên (1)$\Leftrightarrow 2p^{2}-5p+2+5r\geq 0$
*$p\geq 2\Rightarrow 2p^{2}-5p+2+5r=(p-2)(2p-1)+5r\geq 0$(đúng)
*$p\leq 2$.Áp dụng bất đẳng thức Schur ta có:
$r\geq \frac{p(4q-p^{2})}{9}\Leftrightarrow 5r\geq \frac{20pq-5p^{3}}{9}=\frac{20p-5p^{3}}{9}$

$\Rightarrow 2p^{2}-5p+2+5r\geq 2p^{2}-5p+2+\frac{20p-p^{3}}{9}=-\frac{5}{9}p^{3}+2p^{2}-\frac{25}{9}p+2\geq 0$
$\Leftrightarrow 5p^{3}-18p^{2}+25p-18\leq 0\Leftrightarrow (p-2)(5p^{2}-8p+9)\leq 0$(đúng do $p\leq 2$)
Vậy bất đẳng thức được chứng minh
Dấu"=" xảy ra khi a=b=1,c=0 và các hoán vị

#209
nguyenta98

nguyenta98

    Thượng úy

  • Hiệp sỹ
  • 1259 Bài viết

Bài 108:
CMR: \[\left( {1 + {2^2}} \right)\left( {1 + {2^{{2^2}}}} \right)\left( {1 + {2^{{2^3}}}} \right).....\left( {1 + {2^{{2^n}}}} \right) < \frac{1}{3}{.2^{{2^{n + 1}}}}\]

Hôm qua vừa đi học thêm xong mình xin đóng góp một số bài: :icon6:
Bài 92: Cho $a,b,c>0$. CMR:
$Q=\dfrac{\sqrt{ab}}{a+b+2c}+\dfrac{\sqrt{bc}}{b+c+2a}+\dfrac{\sqrt{ca}}{a+c+2b}\le \dfrac{3}{4}$

Bài 108:
Đặt phương trình ở đề bài là $A$
Do vậy $(2^2-1)A=(2^2-1)(2^2+1)(2^{2^2}+1)...(2^{2^{n}}+1) = (2^{2^2}-1)(2^{2^2}+1)(2^{2^3}+1)...(2^{2^{n}}+1)=(2^{2^3}-1)(2^{2^3}+1)...(2^{2^n}+1)=....$
Làm tương tự với hẳng đẳng thức chủ đạo là $a^2-b^2$ ta được $A=\dfrac{1}{3}(2^{2^{n+1}}-1)<\dfrac{1}{3}(2^{2^{n+1}}) \rightarrow Q.E.D$

Bài 92: (của bạn toilaab)
Ta có bất đẳng thức khá quen thuộc $(a+b)(b+c)(c+a)\geq \dfrac{8}{9}(a+b+c)(ab+bc+ca)$ (cái này khá dễ chứng minh dành cho mọi người)
Ta có $\dfrac{1}{(a+b+2c)^2}\le \dfrac{1}{4(a+c)(b+c)}$
Tương tự suy ra $B=\dfrac{1}{(a+b+2c)^2}+\dfrac{1}{(b+c+2a)^2}+\dfrac{1}{(c+a+2b)^2}\le \dfrac{a+b+c}{2(a+b)(b+c)(c+a)}$
Áp dụng bdt trên suy ra:
$B\le \dfrac{a+b+c}{2(a+b)(b+c)(c+a)}\le \dfrac{a+b+c}{(\dfrac{16}{9})(a+b+c)(ab+bc+ca)}$
Suy ra $B\le \dfrac{9}{16(ab+bc+ca)}$
Trở về bài Áp dụng bdt bunhiacopxki suy ra
$Q^2\le (ab+bc+ca)(\dfrac{1}{(a+b+2c)^2}+\dfrac{1}{(b+c+2a)^2}+\dfrac{1}{(c+a+2b)^2})$
Suy ra $Q^2\le (ab+bc+ca)(\dfrac{9}{16(ab+bc+ca)})= (\dfrac{3}{4})^2 \rightarrow Q\le \dfrac{3}{4} \rightarrow Q.E.D$
Dấu $"=" \leftrightarrow a=b=c$

P/S thực ra bài này mình làm được nhờ anh họ gợi ý cho cái bdt trên :icon6:

Bài viết đã được chỉnh sửa nội dung bởi nguyenta98: 18-01-2012 - 15:22


#210
kevinkernpham

kevinkernpham

    Lính mới

  • Thành viên
  • 7 Bài viết
Bài 110: Cho 3 số thực $x,y,z$ không âm thỏa $x+y+z=1$ tìm max của $P=\frac{4(1-x)(1-z)(x+z)}{x+2004y+z}$
____________________________________________

Bạn đánh số thứ tự vào nhé. Lần sau xóa không báo trước

Bài viết đã được chỉnh sửa nội dung bởi Cao Xuân Huy: 18-01-2012 - 16:25


#211
Cao Xuân Huy

Cao Xuân Huy

    Thiếu úy

  • Hiệp sỹ
  • 592 Bài viết

Bài 110: Cho 3 số thực $x,y,z$ không âm thỏa $x+y+z=1$ tìm max của $P=\frac{4(1-x)(1-z)(x+z)}{x+2004y+z}$

Bài 110:
Ta chứng minh: $4(1-x)(1-y)(1-z) \le x+2y+z$
Ta có:
\[VT = 4(x + y)(y + z)(z + x) \le {(x + 2y + z)^2}(z + x) \le (x + 2y + z).\frac{{{{(2x + 2y + 2z)}^2}}}{4} = x + 2y + z\]
Do đó:
\[P \le \frac{{x + 2y + z}}{{x + 2004y + z}} = \frac{{y + 1}}{{2003y + 1}} = \frac{1}{{2003}} + \frac{{2002}}{{2003(2003y + 1)}}\]
Vì $y \ge 0$ nên:
$$P \le \frac{1}{{2003}} + \frac{{2002}}{{2003(2003y + 1)}} \le \frac{1}{{2003}} + \frac{{2002}}{{2003}} = 1 \Rightarrow P_{max}=1$$
Max xảy ra khi:

$\left\{ \begin{array}{l}x = z\\y = 0\\x + y + z = 0\end{array} \right. \Leftrightarrow (x;y;z) = \left( {\frac{1}{2};0;\frac{1}{2}} \right)$

zz

Bài viết đã được chỉnh sửa nội dung bởi Cao Xuân Huy: 19-01-2012 - 23:02

Cao Xuân Huy tự hào là thành viên VMF

Hình đã gửi


#212
kevinkernpham

kevinkernpham

    Lính mới

  • Thành viên
  • 7 Bài viết
Bài 111. cho 2 số thực dương a,b với a+b=1 tìm min P=$(1-\frac{1}{a^{2}})(1-\frac{1}{b^{2}})$

#213
Cao Xuân Huy

Cao Xuân Huy

    Thiếu úy

  • Hiệp sỹ
  • 592 Bài viết

Bài 111. cho 2 số thực dương a,b với a+b=1 tìm min P=$(1-\frac{1}{a^{2}})(1-\frac{1}{b^{2}})$

Xử tiếp. Bài này không khó lắm
Ta có:
\[P = 1 - \frac{1}{{{a^2}}} - \frac{1}{{{b^2}}} + \frac{1}{{{a^2}{b^2}}} = 1 + \frac{{{{(a + b)}^2}}}{{{a^2}{b^2}}} - \frac{{{a^2} + {b^2}}}{{{a^2}{b^2}}} = 1 + \frac{{2ab}}{{{a^2}{b^2}}} = 1 + \frac{2}{{ab}} \ge \frac{8}{{{{(a + b)}^2}}} + 1 = 9\]
Do đó $P_{min}=9 \Leftrightarrow a=b=\frac{1}{2}$

zz

Bài viết đã được chỉnh sửa nội dung bởi Cao Xuân Huy: 19-01-2012 - 23:02

Cao Xuân Huy tự hào là thành viên VMF

Hình đã gửi


#214
Ispectorgadget

Ispectorgadget

    Nothing

  • Quản lý Toán Phổ thông
  • 2946 Bài viết
Bài 112: Cũng dễ :P
Cho x,y,z là các số thực dương thoả $x+y+z=1$. Tìm GTNN của
P=$\frac{(x^3+y^3+z^3)^2+(xyz)^2}{xyz}$

zz

Bài viết đã được chỉnh sửa nội dung bởi Cao Xuân Huy: 19-01-2012 - 23:03

►|| The aim of life is self-development. To realize one's nature perfectly - that is what each of us is here for. ™ ♫


#215
HÀ QUỐC ĐẠT

HÀ QUỐC ĐẠT

    Thượng sĩ

  • Thành viên
  • 295 Bài viết
Từ giả thiết ta có:$x^{3}+y^{3}+z^{3 }\geq \frac{(x+y+z)^{3}}{9}$ và $xyz\leq \frac{1}{27}$
$P=\frac{(x^{3}+y^{3}+z^{3})^{2}}{xyz}+xyz= \sum_{k=1}^{9}\frac{(x^{3}+y^{3}+z^{3})^{2}}{9xyz}+xyz\geq 10\sqrt[10]{\frac{(x^{3}+y^{3}+z^{3})^{18}}{9^{9}(xyz)^{8})}}\geq \frac{10}{27}$
Vậy Min P$=\frac{10}{27}\Leftrightarrow x=y=z=\frac{1}{3}$

Bài viết đã được chỉnh sửa nội dung bởi HÀ QUỐC ĐẠT: 18-01-2012 - 18:11


#216
Đặng Hoài Đức

Đặng Hoài Đức

    Binh nhất

  • Thành viên
  • 36 Bài viết
Bài 113: cho a,b,c >0, a+b+c=3. CMR:
$\frac{ab}{\sqrt[2]{c^{3}+3}}+\frac{bc}{\sqrt[2]{a^{3}+3}}+\frac{ac}{\sqrt[2]{b^{3}+3}}\leq \frac{3}{2}$

Bài viết đã được chỉnh sửa nội dung bởi Đặng Hoài Đức: 18-01-2012 - 18:19


#217
Dung Dang Do

Dung Dang Do

    Dũng Dang Dở

  • Thành viên
  • 524 Bài viết

Với $x=y=z=2$, BĐT sai, bạn xem lại đề nha =.=
Chắc $VP=8xyz$ thì đây lại là một BĐT quen thuộc :D

Xin lỗi em quên mất một điều kiện là $x+y+z=4$.Thật lòng xin lỗi các bạn và Mod. Lần sau em kp tái phạm nữa .Nếu trở về $8xyz$ thì quá đơn giản
@@@@@@@@@@@@

#218
Ispectorgadget

Ispectorgadget

    Nothing

  • Quản lý Toán Phổ thông
  • 2946 Bài viết
Bài 114: Cho các số thực a,b,c thoả mãn $a+b+c=0$. CMR
$ab+2bc+3ca\leq 0$
ĐTTS trường PTNK - ĐHQG TPHCM 2005-2006
Bài 115: Cho a,b,c thực dương thoả mãn $a+b+c=1$. Tìm GTNN của biểu thức
$P=\frac{9}{1-2(ab+bc+ac)}+\frac{2}{abc}$
THTT
_____
Tiện đây mình góp ý cho mấy bạn tham gia topic.
Mọi người ráng đưa lên đây đầy đủ các dạng để việc ôn tập được tốt hơn.
Mọi người có thể đưa các bài thi quốc tế,đại học hoặc TST,... nhưng lời giải không quá phức tạp hoặc biến đổi lằng ngoằng, phải phù hợp trình độ THCS.

zz

Bài viết đã được chỉnh sửa nội dung bởi Cao Xuân Huy: 19-01-2012 - 23:05

►|| The aim of life is self-development. To realize one's nature perfectly - that is what each of us is here for. ™ ♫


#219
Dung Dang Do

Dung Dang Do

    Dũng Dang Dở

  • Thành viên
  • 524 Bài viết
Góp thêm 1 bài nựa:D
Bài 116: Cho $a,b,c>0$, $abc=1$.Chứng minh $BĐT$:
$\frac{a+b+c}{3}\geq\sqrt[5]{\frac{a^{2}+b^{2}+c^{2}}{3}}$
Bài 117: Cho $a,b,c$ ko âm thỏa mãn $a+b+c=1$. Tìm $max$ của biểu thức sau:
$p=\frac{a^{2}b^{2}}{1-ab}+\frac{c^{2}b^{2}}{1-bc}+\frac{a^{2}c^{2}}{1-ac}$
Bài 118:Cho $a,b,c>0$ thỏa mãn $a+b+c=3$. $CMR$
$\sum \frac{5-3bc}{1+a}\geq ab+bc+ac$

Bài viết đã được chỉnh sửa nội dung bởi nguyenta98ka: 18-01-2012 - 20:54

@@@@@@@@@@@@

#220
Dung Dang Do

Dung Dang Do

    Dũng Dang Dở

  • Thành viên
  • 524 Bài viết
Bài 119: Cho $a,b,c>0$ thỏa mãn $abc=1$.Chứng minh rằng:
$\frac{a^{3}}{(1+b)(1+c)}+\frac{b^{3}}{(1+c)(1+a)}+\frac{c^{3}}{(1+a)(1+b)}\geq \frac{3}{4}$
-----------------------------------
Đề thi bán nửa học kỳ I học sinh THCS Yên Thịnh(hơi xoắn tí)
@@@@@@@@@@@@




4 người đang xem chủ đề

0 thành viên, 4 khách, 0 thành viên ẩn danh